Some advertisers offer certain consumers home computers free of charge. Advertisements play continuously on the compu...

Bradkohl on February 10, 2020

Can you explain the issue with B

It seems the promotions are entirely dependent on the internet behavior of those who received free computers.

Reply
Create a free account to read and take part in forum discussions.

Already have an account? log in

shunhe on February 11, 2020

Hi @bradkohl,

Thanks for the question! The problem with (B) is that it uses language that is too extreme. Note that it says that “NO advertisers could offer promotions that give way computers free of charge.” In general, when the LSAT uses this kind of extreme language, you should be wary. It’s not always wrong, but it’s often the case that it makes the answer choice to extreme. Here, the stimulus doesn’t prove that there doesn’t exist somewhere a single advertiser that could offer a promotion that gave away computers free of charge under these circumstances. Perhaps, for example, by giving away free computers, more people would buy their other products. Since (B) is too strong, it’s not the right answer. Hope this helps! Feel free to ask any other questions that you might have.